Difference between revisions of "2017 AIME I Problems/Problem 11"

(Created page with "We know that if <math>5</math> is a median, then <math>5</math> will be the median of the medians. WLOG, assume <math>5</math> is in the upper left corner. One of the two oth...")
 
Line 6: Line 6:
  
 
~Solution by SuperSaiyanOver9000, mathics42
 
~Solution by SuperSaiyanOver9000, mathics42
 +
 +
 +
Solution 2 (Complementary Counting with probability)
 +
 +
Notice that m can only equal 4, 5, or 6, and 4 and 6 are symmetric.
 +
 +
WLOG let <math>m=4</math>
 +
 +
There is a <math>\frac{15}{28}</math> chance that exactly one of 1, 2, 3 is in the same row.
 +
 +
There is a <math>\frac{2}{5}</math> chance that the other two smaller numbers end up in the same row.
 +
 +
<math>9!(1-2*\frac{15}{28}*\frac{2}{5})=362880*\frac{4}{7}=207\boxed{360}</math>.

Revision as of 17:41, 8 March 2017

We know that if $5$ is a median, then $5$ will be the median of the medians.

WLOG, assume $5$ is in the upper left corner. One of the two other values in the top row needs to be below $5$, and the other needs to be above $5$. This can be done in $4\cdot4\cdot2=32$ ways. The other $6$ can be arranged in $6!=720$ ways. Finally, accounting for when $5$ is in every other space, our answer is $32\cdot720\cdot9$. But we only need the last $3$ digits, so $\boxed{360}$ is our answer.

~Solution by SuperSaiyanOver9000, mathics42


Solution 2 (Complementary Counting with probability)

Notice that m can only equal 4, 5, or 6, and 4 and 6 are symmetric.

WLOG let $m=4$

There is a $\frac{15}{28}$ chance that exactly one of 1, 2, 3 is in the same row.

There is a $\frac{2}{5}$ chance that the other two smaller numbers end up in the same row.

$9!(1-2*\frac{15}{28}*\frac{2}{5})=362880*\frac{4}{7}=207\boxed{360}$.